LSAT and Law School Admissions Forum

Get expert LSAT preparation and law school admissions advice from PowerScore Test Preparation.

User avatar
 Dave Killoran
PowerScore Staff
  • PowerScore Staff
  • Posts: 5853
  • Joined: Mar 25, 2011
|
#43247
Complete Question Explanation
(The complete setup for this game can be found here: lsat/viewtopic.php?t=4568)

The correct answer choice is (D)

As mentioned during the setup, because neither F nor G can perform first, L must perform first. Hence, answer choice (D) is correct.

Get the most out of your LSAT Prep Plus subscription.

Analyze and track your performance with our Testing and Analytics Package.